what is the perpendicular and parallel lines of y= -2 -4x

Answers

Answer 1

y=1/4x-2 is a perpendicular line and y=-4x+3 is a parallel line.

The equation y= -2 -4x in the form of slope intercept form y=mx+b is y=-4x-2.

Where m represents the slope and y intercept is -2.

The slope of a line perpendicular to another line is the negative reciprocal of the slope of the given line.

The negative reciprocal of -4 is 1/4.

Therefore, the slope of the perpendicular line is 1/4.

The perpendicular line is y=1/4x-2.

We know that the parallel lines have same slope.

y=-4x+3

Hence, y=1/4x-2 is a perpendicular line and y=-4x+3 is a parallel line.

To learn more on slope of line click:

https://brainly.com/question/16180119

#SPJ1


Related Questions

1.1.5 Which variable represents the independent variable? Provide a reason for your answer. ​

Answers

The determination of the independent variable depends on the specific environment and design of the study or trial. It's the variable that's manipulated or controlled by the experimenter to observe its impact on the dependent variable.

The independent variable is the variable that's manipulated or controlled by the experimenter in an trial. It's the variable that's believed to have an effect on the dependent variable. The dependent variable, on the other hand, is the variable that's being measured or observed to determine any changes or goods caused by the independent variable.

To determine which variable represents the independent variable, we need to consider the environment of the trial or study in question. In numerous cases, the independent variable is designedly manipulated or controlled by the experimenter to observe its impact on the independent variable.

For illustration, in an trial testing the effect of different diseases on factory growth, the type of toxin would be the independent variable as it's designedly manipulated by the experimenter. The dependent variable would be the measured factory growth, as it's the variable being observed and affected by the different diseases.

In summary, the determination of the independent variable depends on the specific environment and design of the study or trial. It's the variable that's manipulated or controlled by the experimenter to observe its impact on the dependent variable.

For more questions on  independent variable

https://brainly.com/question/82796

#SPJ8

Write the next whole number after DB36 in the base-fourteen system?

Answers

Answer:

The next whole number after DB36 in the base-fourteen system would be DB37.

Step-by-step explanation:

In the base-fourteen system, each digit position represents a power of 14. The digits range from 0 to 13, where 0 to 9 represent the numbers 0 to 9, and A to D represent the numbers 10 to 13, respectively.

To determine the next whole number after DB36, we increment the digit in the least significant position. In this case, the least significant position is the units place, represented by the digit 6. Since the next number after 6 is 7, the next whole number in the base-fourteen system is DB37.

Juan quiere hallar la suma de los numeros de 1 hasta n, pero al hacerlo se equivoca y suma dos veces unos de estos numeros, obteniendo como resultado erroneo 100.

Answers

The correct sum of numbers from 1 to n is 100.

We have,

Let's assume the correct sum of numbers from 1 to n is S.

According to the problem,

Juan made a mistake and added one of the numbers twice.

Let's call this number x.

The wrong sum that Juan obtained is 100.

The correct sum, S, can be expressed as the sum of numbers from 1 to n excluding the number x, plus the number x itself:

S = (1 + 2 + 3 + ... + (x-1) + (x+1) + ... + n) + x.

Since Juan added x twice, the wrong sum can be expressed as the sum of numbers from 1 to n without excluding any number: 100 = 1 + 2 + 3 + ... + (x-1) + x + (x+1) + ... + n.

We can subtract the correct sum equation (step 4) from the wrong sum equation (step 5) to find the value of x:

100 - S = (1 + 2 + 3 + ... + (x-1) + x + (x+1) + ... + n) - ((1 + 2 + 3 + ... + (x-1) + (x+1) + ... + n) + x).

Simplifying further, we get: 100 - S = x - x = 0.

From step 6, we see that 100 - S = 0, which means S = 100.

Therefore,

The correct sum of numbers from 1 to n is 100.

In conclusion, the correct sum of the numbers from 1 to n is 100.

Learn more about expressions here:

https://brainly.com/question/3118662

#SPJ1

The complete question:

Juan wants to find the sum of the numbers from 1 to n, but in doing so he makes a mistake and adds one of these numbers twice, obtaining the wrong result 100.

Find the no of 6 digut numbers that can be formed using the digits 1,2,3,4,5,6 once such that the 6 digits nos divisible by its unit digit

Answers

Adding the two cases together, we get a total of 360 + 720 = 1080 six-digit numbers that can be formed using the digits 1, 2, 3, 4, 5, and 6 once, such that the number is divisible by its unit digit.

To find the number of 6-digit numbers that can be formed using the digits 1, 2, 3, 4, 5, and 6 once, such that the 6-digit number is divisible by its unit digit, we can consider the following cases:

Case 1: The unit digit is 2, 4, or 6

In this case, the unit digit is already divisible by itself. We have 3 choices for the unit digit. For the remaining 5 digits, we have 5 choices for the first digit, 4 choices for the second digit, 3 choices for the third digit, 2 choices for the fourth digit, and 1 choice for the fifth digit. Therefore, the total number of 6-digit numbers is 3 * 5 * 4 * 3 * 2 * 1 = 360.

Case 2: The unit digit is 1, 3, or 5

In this case, the unit digit is not divisible by itself. We have 3 choices for the unit digit. For the remaining 5 digits, we have 5 choices for the first digit, 4 choices for the second digit, 3 choices for the third digit, 2 choices for the fourth digit, and 1 choice for the fifth digit. However, for the sixth digit, it cannot be the same as the unit digit since the number needs to be divisible by the unit digit. Therefore, we have 2 choices for the sixth digit. Hence, the total number of 6-digit numbers is 3 * 5 * 4 * 3 * 2 * 2 = 720.

Adding the two cases together, we get a total of 360 + 720 = 1080 six-digit numbers that can be formed using the digits 1, 2, 3, 4, 5, and 6 once, such that the number is divisible by its unit digit.\

For more questions on unit digit

https://brainly.com/question/3643367

#SPJ8

the measures of ABD is (0.2x+52) and the measures of CBD is (0.2x+42) find the value of x

Answers

The value of x in the triangle is determined as 45.

What is the value of x?

The value of x in the triangle is calculated by applying the following formula,.

The measure of angle ABD = 0.2x + 52

The measure of angle CBD = 0.2x + 42

From the diagram, we can set-up the following equations;

x + 16 = 0.2x + 52

Simplify the equation above, by collecting similar terms;

x - 0.2x = 52 - 16

0.8x = 36

Divide both sides of the equation by " 0.8 "

0.8x / 0.8 = 36/0.8

x = 45

Thus, the value of x in the triangle is calculated by equating the appropriate values to each other.

Learn more about angles in triangles here: https://brainly.com/question/22262639

#SPJ1

cos4A/sin2A + sin4A/cos2A = cosec2A​

Answers

We have successfully proven the identity cos(4A)/sin(2A) + sin(4A)/cos(2A) = cosec(2A) Thus, we have shown that the given trigonometric identity is true.

To prove the trigonometric identity: cos(4A)/sin(2A) + sin(4A)/cos(2A) = cosec(2A), we'll work on simplifying both sides of the equation separately.

Starting with the left-hand side (LHS):

LHS = cos(4A)/sin(2A) + sin(4A)/cos(2A)

To simplify this expression, we'll use trigonometric identities and algebraic manipulations.

First, let's rewrite cos(4A) and sin(4A) using double-angle formulas:

cos(4A) = cos^2(2A) - sin^2(2A)

sin(4A) = 2sin(2A)cos(2A)

Substituting these expressions into LHS, we have:

LHS = (cos^2(2A) - sin^2(2A))/sin(2A) + (2sin(2A)cos(2A))/cos(2A)

Now, let's simplify each term separately:

LHS = cos^2(2A)/sin(2A) - sin^2(2A)/sin(2A) + 2sin(2A)cos(2A)/cos(2A)

Using the identity cos^2(2A) = 1 - sin^2(2A), we can rewrite the first term:

LHS = (1 - sin^2(2A))/sin(2A) - sin^2(2A)/sin(2A) + 2sin(2A)cos(2A)/cos(2A)

Now, let's combine the fractions with a common denominator:

LHS = (1 - sin^2(2A) - sin^2(2A) + 2sin(2A)cos(2A))/sin(2A)

Simplifying further:

LHS = (1 - 2sin^2(2A) + 2sin(2A)cos(2A))/sin(2A)

Using the identity 2sin(2A)cos(2A) = sin(4A), we have:

LHS = (1 - 2sin^2(2A) + sin(4A))/sin(2A)

Now, let's simplify the right-hand side (RHS) using the reciprocal identity of sin(2A):

RHS = 1/sin(2A) = cosec(2A)

Since LHS = RHS, we have successfully proven the identity:

cos(4A)/sin(2A) + sin(4A)/cos(2A) = cosec(2A)

Thus, we have shown that the given trigonometric identity is true.

for such more question on trigonometric

https://brainly.com/question/7331447

#SPJ8

(q6) A student wants to find the area of the surface obtained by rotating the curve
, about the x-axis. Which of the following gives the correct area?

Answers

The correct formula to find the area of the surface obtained by rotating the curve, about the x-axis is S = ∫2πy(x) √(1 + (dy/dx)²) dx.

To find the area of the surface obtained by rotating the curve, about the x-axis, we use the formula given below:

S = ∫2πy(x)ds where ds = √(1 + (dy/dx)²) dx

For the curve y = f(x),

we can use the formula given below:

S = ∫2πf(x) √(1 + (dy/dx)²) dx

The student wants to find the area of the surface obtained by rotating the curve, about the x-axis.

So, the correct formula to find the area of the surface is given below:

S = ∫2πy(x)ds = ∫2πy(x) √(1 + (dy/dx)²) dx

Where the curve is y = f(x).

To learn more about : area

https://brainly.com/question/2607596

#SPJ8

Answer:

D) 0.203π square units

Step-by-step explanation:

The area of a surface obtained by rotating the curve about the x-axis in the interval [a, b] is given by the formula:

[tex]\large\boxed{\displaystyle S=\int^{b}_{a}2\pi y\sqrt{1+\left(\dfrac{\text{d}y}{\text{d}x}\right)^2}\; \text{d}x}[/tex]

The given interval is 0 ≤ x ≤ 1. Therefore:

a = 0b = 1

The equation of the curve is:

[tex]y=\dfrac{x^3}{3}=\dfrac{1}{3}x^3[/tex]

Differentiate the function using the following differentiation rule:

[tex]\boxed{\begin{minipage}{4.8 cm}\underline{Differentiating $ax^n$}\\\\If $y=ax^n$, then $\dfrac{\text{d}y}{\text{d}x}=nax^{n-1}$\\\end{minipage}}[/tex]

Therefore:

[tex]\dfrac{\text{d}y}{\text{d}x}=3 \cdot \dfrac{1}{3}x^{3-1}=x^2[/tex]

Therefore, the integral is:

[tex]\displaystyle S=\int^{1}_{0}2\pi \left(\dfrac{1}{3}x^3\right)\sqrt{1+\left(x^2\right)^2}\; \text{d}x[/tex]

[tex]\displaystyle S=\int^{1}_{0} \left(\dfrac{2\pi}{3}x^3\right)\sqrt{1+x^4}\; \text{d}x[/tex]

[tex]\displaystyle S=\dfrac{2\pi}{3}\int^{1}_{0} x^3\sqrt{1+x^4}\; \text{d}x[/tex]

To evaluate the definite integral, use the method of substitution.

[tex]\textsf{Let}\;u=1+x^4[/tex]

Find du/dx and rewrite it so that dx is on its own:

[tex]\dfrac{\text{d}u}{\text{d}x}=4x^3 \implies \text{d}x=\dfrac{1}{4x^3}\; \text{d}u[/tex]

Find the limits in terms of u:

[tex]\textsf{When}\; x = 0 \implies u=1+0^4=1[/tex]

[tex]\textsf{When}\; x = 1 \implies u=1+1^4=2[/tex]

Rewrite the original integral in terms of u and du:

[tex]\displaystyle S=\dfrac{2\pi}{3}\int^{2}_{1} x^3\sqrt{u} \cdot \dfrac{1}{4x^3}\; \text{d}u[/tex]

[tex]\displaystyle S=\dfrac{2\pi}{3}\int^{2}_{1} \dfrac{1}{4}\sqrt{u}\; \text{d}u[/tex]

[tex]\displaystyle S=\dfrac{\pi}{6}\int^{2}_{1} \sqrt{u}\; \text{d}u[/tex]

[tex]\displaystyle S=\dfrac{\pi}{6}\int^{2}_{1} u^{\frac{1}{2}}\; \text{d}u[/tex]

Evaluate the integral using the integration rule:

[tex]\boxed{\begin{minipage}{4 cm}\underline{Integrating $x^n$}\\\\$\displaystyle \int x^n\:\text{d}x=\dfrac{x^{n+1}}{n+1}+\text{C}$\end{minipage}}[/tex]

Therefore:

[tex]\displaystyle S=\dfrac{\pi}{6}\left[\vphantom{\dfrac12}\dfrac{u^{\frac{1}{2}+1}}{\frac{1}{2}+1}\right]^{2}_{1}[/tex]

[tex]\displaystyle S=\dfrac{\pi}{6}\left[\vphantom{\dfrac12}\dfrac{u^{\frac{3}{2}}}{\frac{3}{2}}\right]^{2}_{1}[/tex]

[tex]\displaystyle S=\dfrac{\pi}{6}\left[\vphantom{\dfrac12}\dfrac{2u^{\frac{3}{2}}}{3}\right]^{2}_{1}[/tex]

[tex]\displaystyle S=\dfrac{\pi}{6}\left(\dfrac{2(2)^{\frac{3}{2}}}{3}-\dfrac{2(1)^{\frac{3}{2}}}{3}\right)[/tex]

[tex]\displaystyle S=\dfrac{\pi}{6}\left(\dfrac{4\sqrt{2}}{3}-\dfrac{2}{3}\right)[/tex]

[tex]\displaystyle S=\dfrac{\pi}{6}\left(\dfrac{-2+4\sqrt{2}}{3}\right)[/tex]

[tex]\displaystyle S=\left(\dfrac{-1+2\sqrt{2}}{9}\right)\pi[/tex]

[tex]\displaystyle S=0.203\pi[/tex]

Therefore, from the given answer options, the correct area is 0.203π square units.

You are working on a school newsletter and you need to resize some
pictures. You have a picture that is 5 inches by 7 inches but you need it
to fit into a space that is 10 inches by 14 inches. What scale factor
would change the size of the picture to make it fit the new space?
OA.5
OB. 1/2
OC. 1/4
OD.2

Answers

Scale factor = (10 inches / 5 inches) = 2

D. 2.

prove that the points 2, -1+i√3, -1-i√3 for a equilateral triangle on the argand plane.
units?

Answers

Answer:

Since all three distances are equal to 2√3, we can conclude that the points 2, -1+i√3, -1-i√3 form an equilateral triangle on the Argand plane. This proves the statement.

Step-by-step explanation:

To prove that the points 2, -1+i√3, -1-i√3 form an equilateral triangle on the Argand plane, we need to show that each side of the triangle has the same length, which is equal to the distance between any two of the points.

Let's first find the distance between points 2 and -1+i√3. We can use the distance formula:

d = √[(x₂ - x₁)² + (y₂ - y₁)²]

where (x₁, y₁) = (2, 0) and (x₂, y₂) = (-1, √3). Substituting in the values, we get:

d₁ = √[(-1 - 2)² + (√3 - 0)²] = √[9 + 3] = √12 = 2√3

Next, let's find the distance between points -1+i√3 and -1-i√3:

d = √[(x₂ - x₁)² + (y₂ - y₁)²]

where (x₁, y₁) = (-1, √3) and (x₂, y₂) = (-1, -√3). Substituting in the values, we get:

d₂ = √[(-1 - (-1))² + (-√3 - √3)²] = √[0 + 12] = 2√3

Finally, let's find the distance between points -1-i√3 and 2:

d = √[(x₂ - x₁)² + (y₂ - y₁)²]

where (x₁, y₁) = (-1, -√3) and (x₂, y₂) = (2, 0). Substituting in the values, we get:

d₃ = √[(2 - (-1))² + (0 - (-√3))²] = √[9 + 3] = √12 = 2√3

Since all three distances are equal to 2√3, we can conclude that the points 2, -1+i√3, -1-i√3 form an equilateral triangle on the Argand plane. This proves the statement.

Units were not specified in the question, so the distances are expressed in arbitrary units.

You spin the spinner once, what is P (odd or greater than 3)

Answers

Step-by-step explanation:

P odd = spin of 1 or 3

P greater than 3  = spin of 4  

   three spins out of 4 possible spins   =  3/4

(q17) A geologist finds out that a radioactive substance A that he found in the caves of Africa decays at a rate of 0.03 percent every year. What is the probability that an atom of this substance chosen at random will decay in the next 70 years?

Answers

The probability of an atom of substance A chosen at random decaying in the next 70 years is 2.07%.

The decay rate of a radioactive substance can be calculated using the formula: N=N0e−λt, where N0 is the initial number of radioactive atoms, N is the remaining number of radioactive atoms,

t is the time in years, and λ is the decay constant expressed in inverse years.

The probability of a given radioactive atom decaying during a specific period can be found using the formula: P = 1 - e^(-λt).

In this case, the decay rate of substance A is 0.03% per year, which can be expressed as a decimal fraction of 0.0003.

Therefore, the decay constant λ = 0.0003.

The time period of interest is 70 years.

Therefore, the probability of an atom decaying during this period can be calculated as:

P = 1 - e^(-0.0003*70)P

= 1 - e^(-0.021)P

= 0.0207 or 2.07%

To learn more about : probability

https://brainly.com/question/251701

#SPJ8

..............................................................................

Answers

Answer:

Debbie average daily change is 8 dollars

Step-by-step explanation:

since he lost 56$ this week (7 days) you have to divide 56 by 7 and your average daily change is 8 dollars.

Answer:

A) - 8 dollars

------------------------

Loss in a week is $56, so if we divide it by the number of days, we'll get daily loss amount:

$56/7 = $8

The change is - $8, since we are talking about a loss.

The matching choice is A.

which number produces an irrational answer when multiplied by 0.79?

Answers

Multiplying 0.79 by √2 yields an irrational answer.

To determine which number produces an irrational answer when multiplied by 0.79, we need to identify a number that, when multiplied by 0.79, results in a non-terminating and non-repeating decimal.

If the result of multiplying a number by 0.79 is a rational number, it would have a terminating or repeating decimal representation. However, if the result is an irrational number, it would have a non-repeating and non-terminating decimal representation.

To find such a number, we can look for an irrational number, such as the square root of a non-perfect square. Let's consider √2.

When we multiply 0.79 by √2, the result is:

0.79 * √2 ≈ 1.1141...

The decimal representation of this product does not terminate or repeat, indicating that it is an irrational number. Therefore, multiplying 0.79 by √2 produces an irrational answer.

In summary, multiplying 0.79 by √2 yields an irrational answer.

For more questions on irrational

https://brainly.com/question/30455261

#SPJ8

A company has budgeted 6 2/3 hours to complete a project, with 1/4 of the time spent on research. How much time does the company plan to spend on research? Express your answer as a mixed number.

Answers

Answer:

1 2/3

Step-by-step explanation:

If the company has budgeted 6 2/3 hours to complete a project, and 1/4 of that time is spent on research, we can find the amount of time spent on research as follows:

Total time for the project = 6 2/3 hours

Time spent on research = (1/4) * (6 2/3) hours

We can simplify 6 2/3 to an improper fraction as follows:

6 2/3 = (6 x 3 + 2) / 3 = 20/3

Substituting this value into the equation above, we get:

Time spent on research = (1/4) * (20/3) hours

Multiplying the fractions, we get:

Time spent on research = 5/3 hours

We can convert this improper fraction to a mixed number as follows:

5/3 = 1 2/3

Therefore, the company plans to spend 1 2/3 hours on research.

Suppose you roll a special 10-sided die. What is the probability that the number rolled is a "8" OR a "9"? Enter you answer as a fraction or a decimal with 3 decimal digits.

Answers

Answer:

1/5 or 0.2

-----------------------------

Each number from 1 to 10 has same probability of 1/10.

So we have 1/10 for an '8' and 1/10 for a '9'.

The probability of either '8' or '9' is the sum of the two probabilities:

P(8 or 9) = 1/10 + 1/10 = 2/10 = 1/5 = 0.2

Enter x-values into the table below to determine function values for a function f(x)
for various inputs. Use the function values in order to determine lim f(x).

Answers

The limit (x -> -2) f(x) is equal to -6.

Let's consider the function f(x) = (x² - 4) / (x + 2). We'll calculate the function values for various x-values in the table below and then determine the limit (x -> -2) f(x).

x f(x)

-3 -5

-2.5 -5.25

-2.1 -5.61

-2.01 -5.9602

-2.001 -5.996002

-2.0001 -5.99960002

Now, let's calculate the limit (x -> -2) f(x) using the function values in the table. As x approaches -2, we can observe that the function values approach a specific value:

lim (x -> -2) f(x) ≈ -6

Therefore, the limit (x -> -2) f(x) is equal to -6.

Learn more about Limit Function here:

https://brainly.com/question/7446469

#SPJ1

2. Write the absolute value of the following. a) | -6 -3 | . b) | 0 - 12 |.​

Answers

Answer:

a)9. b) 12

Step-by-step explanation:

a) | -6 -3 | .

-6-3 = -9

|-9| =9

b) | 0 - 12 |.

0-12=-12

|-12|=12

GIVING BRAINLIEST !!!
need help with alg2 homework pleasee, no explanations btw thanks check image below!

Answers

Answer:

1)

A) 0.541 or 0.54(by rounding)

B) 0.675

C)Yes, they are independent because someone doesnt have to attend prom to be a senior and vice versa

2)

a) 0.29

b) 0.04

c) Yes, they are independent because you do not have to live in Long Beach to recommend the provider and vice versa

Step-by-step explanation:

To win the jackpot, 4 different numbers are randomly selected from 1 to 45 and one number from 1 to 30. The order of the first 4 numbers does not matter. What is the probability of winning the jackpot on one try? type your answer as a fraction or in scientific notation.

Answers

The probability of winning the jackpot on one try is 1/30 or 0.0333 (rounded to four decimal places).

To calculate the probability of winning the jackpot on one try, we need to determine the number of favorable outcomes (winning combinations) and the total number of possible outcomes.

The number of favorable outcomes can be calculated by selecting 4 different numbers from 1 to 45 (without considering the order) and one number from 1 to 30. This can be represented as:

C(45, 4) * C(30, 1)

where C(n, r) represents the number of combinations of n items taken r at a time.

The total number of possible outcomes is the total number of ways to select 4 numbers from 1 to 45 (without considering order) multiplied by the number of ways to select one number from 1 to 30. This can be represented as:

C(45, 4) * 30

To calculate the probability, we divide the number of favorable outcomes by the total number of possible outcomes:

Probability = (C(45, 4) * C(30, 1)) / (C(45, 4) * 30)

Simplifying the expression:

Probability = C(30, 1) / 30

Probability = 1 / 30

Therefore, the probability of winning the jackpot on one try is 1/30 or 0.0333 (rounded to four decimal places).

For more questions on probability

https://brainly.com/question/251701

#SPJ8

Please I really need help

Answers

Answer:

the answer is 5 the probability of each colors is 5

An open box is to be constructed from a square piece of sheet metal by removing a square of side 5 feet from each corner and turning up the edges. If the box is to hold 720 cubic feet, what should be the dimensions of the sheet metal?

Answers

Step-by-step explanation:

Let's solve this problem step by step.

1. Start by drawing a diagram to visualize the problem. We have a square piece of sheet metal, and we need to remove squares from each corner to create an open box.

Let's assume the side length of the original square sheet metal is "x" feet.

After removing squares of side length 5 feet from each corner, the remaining dimensions of the sheet metal will be (x - 10) feet.

The height of the box will be 5 feet.

Therefore, the dimensions of the open box will be (x - 10) feet (length and width), 5 feet (height).

2. Use the formula for the volume of a rectangular box to set up an equation:

Volume = Length x Width x Height

Given that the volume is 720 cubic feet, we can write:

(x - 10) * (x - 10) * 5 = 720

Simplifying the equation:

5(x - 10)^2 = 720

3. Solve the equation to find the value of x:

Divide both sides of the equation by 5:

(x - 10)^2 = 144

Take the square root of both sides:

x - 10 = ±12

Solve for x:

x = 10 + 12 or x = 10 - 12

x = 22 or x = -2

Since we are dealing with lengths, we discard the negative value of x.

Therefore, the side length of the original square sheet metal is 22 feet.

4. Finally, calculate the dimensions of the sheet metal:

Length = Width = x - 10 = 22 - 10 = 12 feet

So, the dimensions of the sheet metal are 12 feet by 12 feet.

Kindly Heart and 5 Star this answer, thanks!

Form a sequence that has one arithmetic mean between 35 and 45

Answers

Answer: 40 is the arithmetic mean

Step-by-step explanation:

Kevin was thinking of a number. Kevin adds 5 to it, then doubles it and gets an answer of 48.4. Which of the answers below describes this scenario?

A) x + 5 x 2 = 48.4
B) 10x = 48.4
C) 2(x + 5) = 48.4
D) 48.4 - 5/2 = x

Answers

Answer:

C) 2(x+5) = 48.4

Step-by-step explanation:

Here's what happens each step

1) x

2) x+5

3) 2(x+5) = 48.4

Answer:

The correct answer is C) 2(x + 5) = 48.4.

Step-by-step explanation:

Let's break down the problem step by step:

Kevin was thinking of a number, let's call it x.Kevin adds 5 to it: x + 5Kevin doubles the result: 2(x + 5)The result is 48.4: 2(x + 5) = 48.4

Therefore, the equation that represents this scenario is 2(x + 5) = 48.4.

To solve for x, we can simplify the equation as follows:

2(x + 5) = 48.4

2x + 10 = 48.4

2x = 48.4 - 10

2x = 38.4

x = 19.2

Therefore, the number Kevin was thinking of was 19.2.

#SPJ1

Someone explain what quantum entanglement is with ur own words​

Answers

Answer:

Some points about quantum entanglement in my own words are:

Quantum entanglement is a physical phenomenon that occurs when pairs or groups of particles are generated or interact in ways such that the quantum state of each particle of the pair or group cannot be described independently of the state of the others, even when the particles are separated by a large distance.The topic of quantum entanglement is at the heart of the disparity between classical and quantum physics: entanglement is a primary feature of quantum mechanics lacking in classical mechanics.Quantum entanglement is a highly counterintuitive phenomenon that has profound implications for our understanding of the universe.Quantum entanglement has the potential to be used in a variety of applications, such as quantum computing and quantum communication.Quantum entanglement is a rapidly developing field of research, and scientists are still learning about its many implications.

Here are some additional details about quantum entanglement:

Quantum entanglement is a consequence of the wave-like nature of matter and energy at the quantum level.When two particles are entangled, they share a single quantum state. This means that if you measure the state of one particle, you will instantly know the state of the other particle, no matter how far apart they are.Quantum entanglement is not limited to particles. It can also occur between atoms, molecules, and even larger objects.Quantum entanglement is a very real phenomenon that has been verified in numerous experiments.Quantum entanglement has the potential to revolutionize the way we think about information and communication. It could be used to create new types of computers and communication networks that are far more powerful than anything that exists today.

Quantum entanglement is a fascinating and mysterious phenomenon that is still not fully understood. However, it has the potential to change the world in many ways

The arc length of a sector is equal to one fourth of the radius. Express the arc length s as a function of the area a of the sector.

















The arc length of a sector is equal to one fourth of the radius. Express the arc length s as a function of the area a of the sector.

Answers

In this relationship, the arc length is inversely Proportional to the area. As the area increases, the arc length decreases, and vice versa.

The relationship between the arc length (s) and the area (a) of a sector, let's start by understanding the formulas for each.

The arc length of a sector can be calculated using the formula:

s = θr

where s is the arc length, θ is the central angle of the sector in radians, and r is the radius.

The area of a sector can be calculated using the formula:

a = (1/2)θr^2

where a is the area, θ is the central angle of the sector in radians, and r is the radius.

Given that the arc length is equal to one fourth of the radius, we can express this relationship as:

s = (1/4)r

Now, let's express the central angle (θ) in terms of the area (a).

From the formula for the area of a sector, we can rearrange it to solve for θ:

a = (1/2)θr^2

Multiplying both sides by 2/r^2, we get:

2a/r^2 = θ

Substituting this value of θ back into the equation for the arc length (s), we have:

s = (1/4)r = (1/4)r(2a/r^2) = (1/2a)r

Therefore, we have expressed the arc length (s) as a function of the area (a) of the sector:

s = (1/2a)r

In this relationship, the arc length is inversely proportional to the area. As the area increases, the arc length decreases, and vice versa.

For more questions on Proportional .

https://brainly.com/question/29774220

#SPJ8

3x+4
12. Simplify +
x+2
x²+2x
2x+4

Answers

[tex] \sf \large \frac{3x + 4}{x + 2} + \frac{ {x}^{2} + 2x}{2x + 4} [/tex]

[tex] \sf \large \frac{3x + 4}{x + 2} + \frac{ x(x + 2)}{2(x + 2)} [/tex]

[tex] \sf \large \frac{3x + 4}{x + 2} + \frac{ x \cancel{(x + 2)}}{2 \cancel{(x + 2)}} [/tex]

[tex] \sf \large \frac{3x + 4}{x + 2} + \frac{ x}{2} [/tex]

[tex] \sf \large \frac{2(3x + 4) + x(x + 2)}{2(x + 2)} [/tex]

[tex] \sf \large \frac{6x + 8 + {x}^{2} + 2x}{2x + 4} [/tex]

[tex] \sf \large \frac{ {x}^{2} + 8x + 8}{2x + 4} [/tex]

my first number is -4 and my fifth number is 8 what is the third number

Answers

To find the third number in the sequence given that the first number is -4 and the fifth number is 8, we need to determine the pattern or rule that governs the sequence.

One possible pattern is that the sequence is an arithmetic sequence, where each term is obtained by adding a common difference (d) to the previous term.

Let's calculate the common difference using the first and fifth numbers:

Common difference (d) = 8 - (-4)

= 8 + 4

= 12

Now, we can find the third number by adding the common difference to the first number:

Third number = First number + 2 * Common difference

= -4 + 2 * 12

= -4 + 24

= 20

Therefore, the third number in the sequence is 20.

The first term of a geometric sequence is 15, and the 5th term of the sequence is 243/125.

What are the geometric means between these terms?

Answers

The geometric sequence is:

[tex]\boxed{15,9,\frac{27}{5},\frac{81}{25}, \frac{243}{125} }[/tex]

What is geometric sequence?

A geometric sequence, or geometric progression, is a sequence of numbers where each successive number is the product of the previous number and some constant  [tex]\text{r}[/tex].

Now,

Given that the first term of the geometric sequence is 15The fifth term of the sequence is [tex]\frac{243}{125}[/tex]

We need to find the 2nd, 3rd and 4th term of the geometric sequence. To find these terms, we need to know the common difference. The common difference can be determined using the formula,

[tex]\text{a}_{\text{n}}=\text{a}_1(\text{r})^{\text{n}-1}[/tex]

Where [tex]\text{a}_1=15[/tex] and [tex]\text{a}_5=\frac{243}{125}[/tex]

For [tex]\text{n}=5[/tex], we have,

[tex]\dfrac{243}{125}=15(\text{r})^4[/tex]

Simplifying, we have,

[tex]\sf r= \dfrac{3}{5}[/tex]

Thus, the common difference is [tex]\sf r= \frac{3}{5}[/tex]

Now, we shall find the 2nd, 3rd and 4th terms by substituting [tex]\sf n=2,3,4[/tex] in the formula [tex]\text{a}_{\text{n}}=\text{a}_1(\text{r})^{\text{n}-1}[/tex]

For [tex]\sf n=2[/tex]

[tex]\text{a}_2=15\huge \text(\dfrac{3}{5}\huge \text)^1[/tex]

   [tex]=\bold{{9}}[/tex]

Thus, the 2nd term of the sequence is 9

For [tex]\sf n=3[/tex], we have,

[tex]\text{a}_3=15\huge \text(\dfrac{3}{5}\huge \text)^2[/tex]

[tex]=15\huge \text(\dfrac{9}{25}\huge \text)[/tex]

[tex]\bold{=\dfrac{27}{5}}[/tex]

 

Thus, the 3rd term of the sequence is [tex]\bold{{\frac{27}{5}}}}[/tex]

For [tex]\sf n=4[/tex], we have,

[tex]\text{a}_4=15\huge \text(\dfrac{3}{5}\huge \text)^3[/tex]

[tex]=15\huge \text(\dfrac{27}{25}\huge \text)[/tex]

[tex]\bold{=\dfrac{81}{25}}[/tex]

   

Thus, the 4th term of the sequence is [tex]\bold{\frac{81}{25}}[/tex]

Therefore, the geometric sequence is:

[tex]\boxed{\bold{15,9,\frac{27}{5},\frac{81}{25}, \frac{243}{125}}}[/tex]

Learn more about geometric sequence at:

https://brainly.com/question/32003121

Triangle XYZ - Triangle JKL. Use the image to answer the question. Determine the measurement of KL.

A. KL=11.99
B. KL=10.66
C. KL=10.14
D. KL=10.01

Answers

Answer:

  C.  KL = 10.14

Step-by-step explanation:

You want the length of segment KL in ∆JKL given it is similar to ∆XYZ with lengths JK=11.31, XY=8.7, YZ=7.8.

Similar triangles

Corresponding sides of similar triangles are proportional. It can be useful to identify corresponding sides and their given measures:

  JK = 11.31, XY = 8.7

  KL = ?, YZ = 7.8

This lets us write the proportion ...

  KL/JK = YZ/XY

  KL/11.31 = 7.8/8.7 . . . . . . . . . . use given values

  KL = 11.31(7.8/8.7) = 10.14 . . . multiply by 11.31

The measurement of KL is 10.14 untis.

__

Additional comment

You have to go by the sequence of vertices in the similarity statement, not the appearance of the figure. One triangle is rotated from the other in the figure, so that parallel sides are not corresponding sides.

<95141404393>

what is the product: -4[ 8 -1 -5 9
A: [-32 4 20 -36] B: [32 -4 -20 36] C: [4 -5 -9 5] C: [-32 -1 -5 9]

Answers

Answer:

A

Step-by-step explanation:

To find the product of -4 and the vector [8, -1, -5, 9], you need to multiply each element of the vector by -4. Here's the calculation:

-4 * 8 = -32

-4 * -1 = 4

-4 * -5 = 20

-4 * 9 = -36

Therefore, the product of -4 and [8, -1, -5, 9] is:

[-32, 4, 20, -36]

So the correct answer is A: [-32, 4, 20, -36].

Other Questions
Mr. Blah and Mrs. Bleh, are friends and they met at university when they were studying Business Law at La Salle. They share a passion for virtual reality and virtual games. After completing their studies, they decided to start up a new business based on Mr. Blah's idea regarding a virtual reality without glasses. Mr. Blah's idea consists of a very powerful projector with a lot of speakers of different sizes and overall software that changes the video and the involving sound taking into account upon user's viewing. There is also a very incipient augmented virtual reality. A hedge funda.Is available only to invited, normally wealthy, investorsb.Invests in commercial or residential propertyc.Invests in bonds issued by companies which have financialproblemsd.Inve If an employee has a Form W-4 from 2019 or earlier on file with their employer: C-1 Calculate the observed risk premium in each year for the large-company stocks versus the T-bills. What was the arithmetic average risk premium over this period? (A negative answer should be indicated by a minus sign. Do not round intermediate calculations and enter your answer as a percent rounded to 2 decimal places, e.g., 32.16.) c-2 Calculate the observed risk premium in each year for the large-company stocks versus the T-bills. What was the standard deviation of the risk premium over this period? (Do not round intermediate calculations and enter your answer as a percent rounded to 2 decimal places, e.g., 32.16.) C-1. C-2 Average risk premium Standard deviation Consider the following rates of return: US Treasury Bill Large- Year Company Stocks 3.96% 14.12 19.01 -14.67 -32.16 6 37.26 4.50% 4.88 3.80 6.96 4.88 6.14 a. Calculate the arithmetic average returns for large-company stocks and T-bills over this period. (Do not round intermediate calculations and enter your answers as a percent rounded to 2 decimal places, e.g., 32.16.) Large company stocks T-bills b. Calculate the standard deviation of the returns for large-company stocks and T-bills over this period. (Do not round intermediate calculations and enter your answers as a percent rounded to 2 decimal places, e.g., 32.16.) Large company stocks T-bills 1 Newlyweds Jamie Lee and Ross have had several milestones in the past year. They are newly married, recently purchased their first home, and now have twins on the way! Jamie Lee and Ross have to seri From the properties of the regression line, show that a) = b) ; ; = 0 I need help creating a thesis statement & 3 supporting key points based on the article " Eat Turkey, Become American " by Marie Wyung-Ok Lee which characteristics of organisms in a population would indicate the highest potential for evolutionary change (214) 1. Distinguish between technical efficiency and allocative efficiency. Use the two concepts of efficiency to compare a perfect market structure with a monopoly. statically indefenders. connected in parallel and are The identical pumps wolking. The age The system operates if at least one pampo is specific failure rated this type of pump is of constant value, 2 d 0.01 calculate the probability of survival ater 100 hours of single pump? per hour. These proces components are related in seves. This if any one of the components falls, the enture system fails. calculate the reliability and probabelety of each component is given, calculate the overall reliability, overall probability, overall failure late and MTBF of the system. failu rate us component facere Probabilly P = I-R Relrabilty R = out (faults/ y^) Contul value 0. 60 0.75 0.25 controller 0.29 DD cell 0.24 1.41 0.76 Shane Company uses an accelerated depreciation method for income tax purposes and the straight-line depreciation method for financial reporting purposes. As of December 31, 2014, Shane has a deferred tax liability balance related to depreciation temporary differences of $80,000. In 2015, depreciation for income tax purposes was $360,000, while depreciation for financial reporting purposes was $300,000. If the income tax rate is 30%, no other temporary or permanent differences exist, and taxable income is $400,000, the entry to record income tax expense on December 31, 2015, would include adebit to Income Tax Expense for $138,000credit to Income Taxes Payable for $138,000debit to Deferred Tax Asset for $120,000credit to Deferred Tax Liability for $120,000 A licensed producer worked independently for 20 years selling insurance as an appointee of several insurance companies. If the producer chose not to renew the license, the producer mayA. receive renewal or other deferred commissions, since he was licensed at the time of the salesB. continue to sell additional insurance to his existing clients, but at a reduced commission levelC. be paid commission for soliciting or negotiating insurance contracts, but not for new salesD. no longer receive renewal commissions. A food manufacturer is trying to maximize profit by selling wheat-based cereal (C) and wheat bread(B) with raw wheat (W). The production functions are: Cereal: C = 23Wc-1.5Wc Bread: B = 64WB - 2WB Constraint: Wc+WB = 9.157Profit is $1.00 per box of cereal and $0.50 per pack of wheat bread. There are 4,086 units of raw wheat available. How much wheat should go to the cereal (We)? Enter as a value. ROUND TO THE NEAREST WHOLE NUMBER. Sally Rubber Co. is looking at investing in a production facility that will require an initial investment of $500,000. The facility will have a three-year useful life, and it will not have any salvage value at the end of the projects life. If demand is strong, the facility will be able to generate annual cash flows of $250,000, but if demand turns out to be weak, the facility will generate annual cash flows of only $125,000. Sally Rubber Co. thinks that there is a 50% chance that demand will be strong and a 50% chance that demand will be weak.If the company uses a project cost of capital of 13%, what will be the expected net present value (NPV) of this project?-$60,148-$42,963-$57,284-$40,099 When you calculated the RNF for your company you found that Liabilities and Owner's Equity were $50,000 greater than assets. Your CFO Dom Perinion asks what this means for the company? Explain the results of your RNF calculation. Suppose that sin()=1/8. What cos()=_________ The difference in the values of American and European puts on futures when the exercise price is 100, the futures price is 105.1271, the interest is 5%, the annualized standard deviation is 30%, and the options expire in one year. Is higher when the interest rate is 5% than when it is 1%. Is lower when the interest rate is 5% than when it is 1% The same for a 5% or 1%interest rates Urban sprawl is related to O a. Central city decline O b. Improved transportation OC. Edge city decline O d. Both 'a' and 'b' Oe. All of the above ('a', 'b', and 'c') The Schmitty family loves pickled raspberries, honey clusters and candied almonds. The family's pr-hc-ca utility function is given byU(x, y, z) = 5 ln x + 6 ln y + 9 ln z where x = the number of kilograms of pickled raspberries, y = the number of kilograms of honey clusters and z = the number of kilograms of candied almonds that they consume each month. Pickled raspberries cost $6/kg, honey clusters cost $9/kg and candied almonds cost $18/kg. (a) Use the method of Lagrange multipliers to find the quantities of pickled raspberries, honey clusters and candied almonds that the Schmitty family should consume each month to maximize their utility, given that the monthly budget for these goods is $720. What is their maximum monthly utility? (b) Repeat part (a) in the case that the pr-hc-ca budget is given as 3, and express the Schmittys' maximum monthly pr-hc-ca utility in the form U* (B) = A ln - B. In other words, find the constants A and B. what did the first quartering act allow british soldiers to do?